• Aucun résultat trouvé

! o o r z > x = 0 f o r z = 0 i s c h a n g e d i n t o ~ - z 0 f o r z = ~ o ( x , y ) , < 0 f o r z < 0 , 0 f o r

N/A
N/A
Protected

Academic year: 2022

Partager "! o o r z > x = 0 f o r z = 0 i s c h a n g e d i n t o ~ - z 0 f o r z = ~ o ( x , y ) , < 0 f o r z < 0 , 0 f o r"

Copied!
23
0
0

Texte intégral

(1)

ARKIV FOR MATEMATIK Band 6 nr 22

1.65 12 1 Communicated 13 October 1965 b y L. GXI~DING and L. CARLESO~

M i n i m i z a t i o n p r o b l e m s f o r t h e f u n c t i o n a l

supxF(x,f (x),f'(x)). (II)

By Gu~NAR AnoNsso~

Introduction

The present paper is a continuation of the paper [1]. This means t h a t we shall t r y to minimize the functional

/t(/) =sup~/~(z,/(x),/'(x))

over the class :~ of all absolutely continuous functions/(x) which satisfy the boundary conditions/@1)

=Yl

and/(Xz) =Y2-

I n Chapter 1 we consider the question of the existence of a minimizing function, which was left open in [1].

I n Chapter 2 some of the previous results are generalized to a wider class of func- tions

F(x,y,z):

the previous condition

~F

~z

> 0 for z > 0

[!o orz> x

= 0 for z = 0 is changed into ~-z 0 for z = ~o(x, y),

< 0 for z < 0 , 0 for

z<o:,(x,y).

An existence proof for absolutely minimizing functions is given in the general case.

I n Chapter 3 we examine further the properties of a.s. minimals. We also consider uniqueness questions for a.s. minimals and minimizing functions. For reasons of simplicity, this chapter is restricted to the case

o(x,y)=-0.

I n Chapter 4, finally, we give some examples in order to illustrate the previous exposition.

We shall now introduce notations for some conditions on

F(x,y, z).

They are as- sumed to hold for x E J and for all real y and z. Here J is an interval which will be stated explicitly in most cases. If it is not explicitly given, then the choice of J will be clear from the context.

1. F(x,y,z)

is continuous and

~F/~z

exists.

~F(x, y, z)

is

2A. ~z

> 0 if

= 0 if

< 0 if z > 0 , Z ~ 0 , z < 0 .

409

(2)

c. AROrCSSO~,

Minimization problems. I I

2 B . There is a continuous f u n c t i o n

co(x, y)

such t h a t

OF(x, y, z)

Oz

f!

O if

z>o~(x,y),

is 0 if z = o ~ ( x , y ) , 0 if

z<co(x,y).

3. liml~l_,~r

F(x,

y, z ) = + oo if x a n d y are fixed.

T h e conditions 1 a n d 3 will be assumed to hold t h r o u g h o u t this paper, t o g e t h e r with 2 A or 2 B . 1 2 A is a s s u m e d to hold in C h a p t e r 1 a n d Chapter 3; in Chapter 2 we shall use t h e m o r e general condition 2 B. Xt is easy t o see (using t h e H e i n e - B o r e l covering theorem) t h a t

lim ( inf

F(x, y, z))= +

I z l ~ ~x~<~

l y K K

for every c o m p a c t interval [:r c J a n d for every K > 0.

Chapter 1. The question of the existence of a minimizing function T h e assumptions on

F(x,y,z)

in this c h a p t e r are m a i n l y t h e same as in [1]. W e shall assume t h a t t h e following conditions are satisfied t h r o u g h o u t t h e c h a p t e r : 1, 2 A a n d 3. T h e interval J will be chosen as

Xl<~X<~X ~.

1. W e shall n o w give an example which shows t h a t t h e existence of a minimizing function does n o t follow f r o m these properties of

F(x, y, z).

A counterexample.

Choose

F(x, y, z) =z2/(y 2 +

1) 2 - cosx - x ( 2 ~ - x ) c o s x - ~ ( y ) (2~ - ~ ( y ) ) ,

where~(y)=arctgy+~/2;

x l = 0 a n d

I x2=2~'

Yl = 0 [ Y2 = 0.

L e t /E :~. W e h a v e

H(I)

>I F ( ~ , / ( ~ ) , 0) = 1 + ~ 2 - ~ ( / ( ~ ) ) [ 2 ~ - ~ ( / ( ~ ) ) ] > 1,

since 0 < ~ ( / ( ~ ) ) < z . H e n c e H ( / ) > I for all /E:~. Suppose n o w t h a t 0 < ~ < g / 2 a n d consider t h e function

1 T h e r e is one exception: E x a m p l e 5 in C h a p t e r 4.

(3)

ARKIV FOR MATEMATIK. Bd 6 nr 22 tg x

tg (2 :~ - x )

I t is easy to verify t h a t

for 0 ~ < x - ~ - ~, for 2 - - (~ ~ < x ~ + (5, for 3 - ~ + ~ ~<x~<27~.

H(g~) = F ( z , g~(ze), O) = 1 + ~2.

This proves t h a t infr~ ~ H ( / ) = 1 and t h a t there is no minimizing function. Compare the r e m a r k to Theorem 1.1.

2. I t follows from this example that, in order to ensure the existence of a mini- mizing function, we m u s t impose suitable extra conditions on F(x,y,z). We shall describe a few such conditions (they are separately sufficient), but we are not aiming at a complete discussion of the question.

I n each existence proof, we shall need

Lemma 1.1. Let {/~(x)}~ be a sequence o//unctions in :~ such that supl.<~<~ H(/~) <

and suppose that ]~(x) -+/(x) uni/ormly /or x I <<. x <~ x 2 . Then / 6 :~ and H(/) <~ lim~_~ ~ H(/r).l Proo/. Since ]/r(x)] ~<K1 a n d H(/~)<~K 2 for all v, it follows from the properties of F t h a t ]/:(x)] ~<K a. Hence all/~(x) a n d / ( x ) satisfy a uniform Lipschitz condition.

Clearly /6:~, and the rest of the proof follows easily. (Use the fact t h a t if /'(xo) exists, then we have

~(xo, /(xo), z) >~ F(Xo, /(xo), /'(xo))

for all z >~/'(xo) or for all z <<./'(xo), together with a continuity argument.)

L e t us now return to the minimization problem and introduce the notation M 0 =inf~E~ H(/). Suppose t h a t {/~}F is a minimizing sequence, i.e. lim~_~ ~ H ( / , ) = M 0.

I f the f u n c t i o n s / , ( x ) are uni/ormly bounded, then it follows (as above) t h a t t h e y are equicontinuous, and then there is a uniformly convergent subsequence. L e t the limit function b e / ( x ) . I t follows from the l e m m a t h a t H ( / ) = M 0.

Hence, in order to prove t h a t there is a minimizing function, it is sufficient to prove t h a t there is a minimizing sequence of uniformly bounded functions.

I n m a n y cases, it can be seen from the function F(x, y, 0) t h a t there is a minimizing function. This possibility is illustrated b y the following theorem.

Theorem 1.1. Suppose that there is a number Y1 >~max (Yl, Y2) such that m a x F(x, Yl, O) = m a x (F(~, Yl, 0), F(/~, Y1, 0))

/or all :r and fi satis/ying xl <~<fl<~x ~. Suppose also that there is a Y2 ~< rain (Yl, Y2) with the same property. Then there is a minimizing/unction.

number 1 In other words, the functional H(]) is lower semi-continuous.

411

(4)

G. AROrr

Minimization problems. I I

Proo/.

Y1 if / ( x ) > Y1, L e t / ( x ) e ~ and form

g(x)

= /(x) if

Y2 <~/(x) <~ Y1,

[Y2 if

/(x)<Y2.

Clearly

H(g)<.H(/)

and the rest of the proof is obvious.

I t is not difficult to find different conditions on

F(x,

y, 0) which lead to the same result. (For instance,

F(x,

y, 0) increasing in y for y >/]71 and all x, decreasing in y for Y ~< Ye and all x, are also sufficient conditions for the existence of a minimizing function.)

3. Suppose that there is a number

M > M o

(M0=infrE~

H(/))

such t h a t

H ( / ) < M

implies t h a t max

I/(x) l<~g,

where K depends on M but not on/(x). Then, clearly, every minimizing sequence is bounded and there is a minimizing function.

I t is evident t h a t every number M > M 0 has this property if limlzl_~

F(x, y, z) = +

uniformly for all x and y. But there m a y exist numbers M with this property even if the limit is not uniform. We can describe the result of this section thus: if the roots zl and z2 of the equation

F(x, y, z ) = M

do not increase too fast in absolute value when [Yl -+c~, then the attainable set

E(M)

is bounded, and then there is a mini- mizing function. For instance, it will follow from Theorem 1.2' t h a t there is a minimizing function if liml~l_~r

F(x,

y, y) = + ~ uniformly in x.

Let us denote b y

El(M)

the set of points (x, y) such that

a) xl<~x<~x~,

b) there is an absolutely continuous function

g(t),

joining the points @1, Yl) a n d

(x,y),

such t h a t

sup

F(t, g(t), g'(t)) <~ M.

t

The set

Ee(M )

is defined analogously, but with

(x2,y~)

instead of

(xl, Yl).

We are

interested in the set

E(M) = EI(M ) N E2(M ).

I t is clear that

H(/) <~ M

implies t h a t max

I/(x) I <~ K

if and only if

E(M)

is bounded, and in t h a t case the constant K is determined by

E(M).

We shall first examine

E(M)

in the case

F = F(y, z).

Later, we shall t r y to carry over some of the results to the general case.

Using some results from [1], we can easily prove the following theorem (concerning

dPM(t), ~pM(t)

and the integrals below, see section 2 of [1]):

Theorem 1.2.

Assume that F is independent o / x and that M > Mo.

A)

E(M) is unbounded above (i.e. it contains points with arbitrarily large ordinates) i/and only i/the integrals

f ~ dt and 12= f~ dt

11

J~, O ~ ( t ) J ~ -

~,(t)

are well-de/ined, convergent and

I i + I ~ < x 2 - x 1.

(1)

(5)

A R K I V FOR M A T E M A T I K . B d 6 n r 2 2

B)

E(M) is unbounded below i / a n d only i/the integrals

i3=f _ - V,~(t) dt and i4=fu _ opt(t) dt are well-de/ined, convergent and

I3 + I4 < x 2 - x 1.

(2)

Proo].

We restrict the proof to the first p a r t of the theorem. Assume first t h a t

B(M)

is unbounded above. L e t (x0, Yo) C

E(M)

and Yo > m a x

(Yl,

Y2). I t follows from Theorem 2 in [1] t h a t

f yV~ dt

1 ~ ~< x ~ xl a n d

f y . dt

~<x 2 - x 0.

, - ~ , ~ ( t )

I f we add these inequalities and let Y0 tend to infinity, then we have proved one p a r t of the assertion.

Next, suppose t h a t

I i + I 2 < x 2 - x 1.

I t follows from the discussion of " t h e at- tainable cone" in section 2B of [1] t h a t

(x, y ) E E I ( M )

if x 1 + I l < x < x 2 and

Y>~Yl.

Similarly, it follows t h a t

(x, y)EE2(M )

if

x 1 <x<~x 2 - I 3

and

Y>~Y2.

Clearly, all points (Xl + I1, y), where y ~> m a x (Yl, Y2), belong to

E(M).

This completes the proof of Theorem 1.2, since the reasoning in the case B is analogous.

A consequence of the theorem is t h a t

E(M)

is bounded if a n d only if neither (1) nor (2) is satisfied.

L e t us now consider the general case

F = F ( x , y, z).

I t can be

"reduced"

to the previous case simply b y neglecting the dependence on x. This m a y be considered to be a crude method, but, nevertheless, it works in m a n y cases.

The functions

~gM(y)

and

~fM(Y)

w e r e defined in [1]. Clearly, we can introduce here the corresponding functions

O(x,y,M)

and ~(x, y, M). Now fix y and M and suppose t h a t U =

{x IF(x,

y, 0) ~<M} is non-empty.

De/inition: I ~(y'

M) = max~

~ v O(x, y, M),

[ fl(y,

M) = minx ~ v ~f(x, y, M).

I t is easy to verify t h a t these functions are continuous and t h a t they are monotonic in M, as are (I)~(y) and ~v~(y). Finally, an argumentation, similar to the one used in the special case F =

F(y,z),

leads to the following result:

Theorem 1.2'.

Assume that M > M o.

A)

I / E ( M ) is unbounded above, then the integrals dt

I1= , a(t,M) and

dt 12=

~ - f l ( t , M )

are well.de/ined, convergent and I 1 + 12 <<-x2 -x l .

B)

I / E ( M ) is unbounded below, then the integrals

413

(6)

c. ARO~SSOr~, Minimization problems. I I

i3=f dt

_ - f l ( t , M ) and Y~ dt

I ~ = _ a(t, M ) are well-defined, convergent and I a + I a <~ x 2 - x 1,

An application of this theorem is given in Example 1 in Chapter 4. Compare also the counterexample.

Chapter 2. A more general case. P r o o f of the existence of absolutely minimizing functions

]. I n this section we shall generalize some of the previous results to a more general class of functions F(x, y, z).

We are not going to change the condition limN_~cF(x, y, z ) = + c o , which has turned out to be very useful and which is a rather natural condition. However, the assumption t h a t F(Xo, Yo, z) has a minimum at z = 0 for all (x0, Y0), is more restrictive, and we shall assume instead t h a t F(x0, Yo, z) has a minimum at z=eo(x0,yo) , where oo(x,y) is a continuous function. Compare Example 5 in Chapter 4.

Thus, let us assume t h a t the following conditions hold throughout this chapter:

F(x, y, z) E C1; 2B and 3.

The minimization problem is meaningful, since we have H(/) >~ m a x [F(Xl, Yl, O~(Xl, Yl)), F(x2, Y2, c~

for every admissible function/(x).

We shall now carry over some of the results of the paper [1] and of the previous chapter to this more general case.

The analogues of the Theorems 5 and 6 in [1] are Theorem 5'. L e t / ( x ) be an admissible/unction such t h a t : a) /'(x) is continuous on I (x 1 <~x <~x2),

b)

F z ( x , / ( x ) , / ' ( x ) ) > ~ O on I , o r < O on I ,

c) F(x,/(x),/'(x))

= M on I .

T h e n / ( x ) is a m i n i m i z i n g / u n c t i o n 1 (not necessarily unique).

Theorem 6'. Suppose that/(x) E ~ satisfies:

a) /'(x) is continuous on I (x 1 ~x<~x~), b) F z ( x , / ( x ) , / ' ( x ) ) # 0 on I ,

c) F(x,/(x),/'(x))

= M on I .

T h e n / ( x ) is a unique m i n i m i z i n g / u n c t i o n . 1

The proofs are analogous to those of the case ~o(x, y)=~0 and they are omitted.

Lemma 4 in [1] holds in this case without changes. The proof is simple.

1 It also follows that ](x) is an a.s. minimal on I.

(7)

ARKIV FOR MATEMATIK, B d 6 n r 22 A c o n s e q u e n c e of t h i s l e m m a is t h a t we can n e g l e c t sets of m e a s u r e zero. This m e a n s t h a t we can j o i n t w o f u n c t i o n s a t a f i n i t e n u m b e r of p o i n t s w h e r e t h e y a r e e q u a l b y t a k i n g one or t h e o t h e r on successive i n t e r v a l s . T h e n t h e s e p o i n t s will n o t affect t h e v a l u e of t h e f u n c t i o n a l for t h e r e s u l t i n g f u n c t i o n . (This s i t u a t i o n will o c c u r in t h e n e x t section.)

:Next, we m a k e a j u m p t o T h e o r e m 9, w h i c h is c a r r i e d o v e r w i t h o u t changes:

T h e o r e m 9'. I / ] ( x ) is an a.s. minimal on the interval 1 (x 1 ~ x <~x2), then:

1) /(x)EC 1 on I,

2) the differential equation

dF(~,/(x),/'(~))

Fz(X, / ( x ) , / ' ( x ) ) = 0 dx

is satisfied on I in the sense that i / F ~ ( x , / ( x ) , / ' ( x ) ) =#0 on a sub-interval I 1 c I, then F ( x , /(x), /'(x)) is constant on 11.

W e shall p r o v e t h e a s s e r t i o n s 1) a n d 2) u n d e r t h e following, w e a k e r a s s u m p t i o n s

on/(x):

I) /(x) satisfies a L i p s c h i t z c o n d i t i o n on [Xl, x2],

I I ) t h e r e is a c o n s t a n t K > 0 such t h a t if [tl, t2] is a n a r b i t r a r y s u b - i n t e r v a l of I , t h e n t h e r e a r e two (not n e c e s s a r i l y different) m i n i m i z i n g f u n c t i o n s u(x) a n d v(x) be- t w e e n ( t i , / ( t i ) ) a n d (t2,/(t2) ) such t h a t [u(x)[ ~ K , Iv(x)] ~ K a n d u(x) <~/(x) <~v(x) for t 1 ~< x ~< t 2.

(These a s s u m p t i o n s a r e s a t i s f i e d if /(x) is a n a.s. m i n i m a l , since we c a n choose K = m a x z ~ ]/(x) l a n d u ( x ) = v ( x ) = / ( x ) in t h a t case.)

T h e r e a s o n w h y we shall p r o v e t h i s m o r e g e n e r a l r e s u l t is t h a t i t will be useful in t h e n e x t section.

Proo]. 1 Consider a n a r b i t r a r y p o i n t x o on t h e i n t e r v a l x 1 ~ x < x 2 a n d s u p p o s e t h a t t h e u p p e r a n d lower r i g h t - h a n d d e r i v a t i v e s of /(x), :r a n d /~, a r e d i f f e r e n t a t x 0.

(Clearly, t h e y a r e finite.) Choose a n u m b e r 7 ~:o)(Xo,/(Xo)) such t h a t g > ~ >ft. T h e i n i t i a l - v a l u e p r o b l e m F(x, g(x), g'(x))=constant, g(xo)=/(x0), g'(xo)=~, h a s a solu- t i o n ~(x) in a n e i g h b o u r h o o d of x 0. Since ~ > ~ >fi, t h e r e a r e p o i n t s x > x o, a r b i t r a r i l y close t o x0, w h e r e /(x)=~0(x). B u t since ~, ~=eo(x0,/(xo)), i t follows f r o m t h e condi- t i o n I I a n d f r o m T h e o r e m 6' t h a t / ( x ) =q~(x) on [Xo, x0+~t ] for s o m e ~ > 0 . T h e con- t r a d i c t i o n shows t h a t / ( x ) h a s a r i g h t - h a n d d e r i v a t i v e . S i m i l a r l y , i t follows t h a t / ( x ) h a s a l e f t - h a n d d e r i v a t i v e for x 1 < x ~<x 2.

O u r n e x t s t e p is to p r o v e t h a t t h e o n e - s i d e d d e r i v a t i v e s a r e equal. Consider a p o i n t x0, a n d a s s u m e t h a t o:=/'(xo)+4fi=/'(xo)-. Choose a n u m b e r y b e t w e e n a n d ft. C l e a r l y

F(x0,/(Xo), y) < m a x [F(xo,/(xo) , o:), F ( x o, ](Xo), fl)].

S e l e c t a sequence of i n t e r v a l s [Pn, q~] such t h a t p~ <xo <q~, q~-p~-->O

a n d /(q~) - / ( P ~ )

q~ - p~

1 The reasoning will be similar to the one used in [1], but not identically the same.

2s: 4 415

(8)

G. ARONSSO~,

Minimization problems. II

I t is obvious t h a t

lim

M(pn, qn; /(P,), /(q,)) < F(xo,/(xo), ~).

n ---> o v

(The n o t a t i o n M(...) was i n t r o d u c e d in [1].) L e t us assume, for example, t h a t g > y > f l a n d

F(xo,/(xo) ,

~) > F(x0,/(xo)

,

y). L e t the functions

Un(X)

correspond to t h e intervals [p~, q~] according to I I . I t follows t h a t lim~_~r (SUpx

u'~(x))>~ ~z.

Thus

Consequently

lira

H(un) ~ F(xo, /(xo), ~).

n ---> ~ r

lim

H(u~)

> lira

M(pn, qn; /(Pn), /(qn)),

n .--> o r n ---> ~

which is a contradiction.

The reasoning is analogous in the other cases. This proves t h a t / ' ( x ) exists.

Suppose n o w t h a t

Fz(xo,/(xo) ,/'(xo) ) #0.

T h e n we assert t h a t / ( x )

E C 2

in a neigh- b o u r h o o d U of x 0 a n d t h a t

F(x, /(x), /'(x))

is c o n s t a n t in U. This is p r o v e d in t h e same w a y as T h e o r e m 8 in [1]. W e omit the details.

N o w t h e only thing left to prove is t h a t / ' ( x ) is continuous a t those points where

Fz(X, /(x), /'(x))

= 0 . Assume t h e n t h a t

]'(Xo)=o~(x

o,/(x0) ). Assume f u r t h e r t h a t there are n u m b e r s ~n->x0, ~ > x 0 such t h a t

/'(~)>~/'(Xo)+~.

I f the functions

Un(X)cor-

respond to the intervals x 0 ~ x ~ according to I I , t h e n we h a v e sup~

u'~(x) >~/'(xo) § ~.

H e n c e l i m n ~

H(Un) ~ F(Xo,/(Xo) ,/'(Xo)

+~). B u t this contradicts the obvious relation lim

M(xo, ~n; /(xo), /(~n)) <~ F(xo, /(Xo), /'(Xo)).

n - - > o ~

T h e other cases can be t r e a t e d similarly. This completes the proof.

W e h a v e t h u s established some i m p o r t a n t properties of a.s. minimals. B u t we h a v e n o t e x a m i n e d the properties of minimizing functions in general. T h e reason for this is evident f r o m E x a m p l e 2 in Chapter 4.

I n t h e previous c h a p t e r we t r e a t e d the question of the existence of a minimizing function for the case

co(x,y)~O.

I t is easy to verify t h a t L e m m a 1.1 holds in t h e present case. This means t h a t if there is a minimizing sequence of u n i f o r m l y b o u n d e d functions, t h e n there is a minimizing function. I n particular, this is t r u e if

E(M)

is b o u n d e d for some

M > M o . 1

W e shall n o t discuss generalizations of the other criteria in Chapter 1.

2. A problem which we h a v e n o t y e t discussed, is t h a t of t h e existence of absolutely minimizing functions. W e shall give an existence proof for the general case.

Theorem 2.1.

Assume that F(x, y, z) E C 1 and satisfies the conditions 2 B and 3 / o r X I <~x<~X 2 and all y, z. (Thus J=[X1,X2]. )

Assume/urther that, /or every choice o/(~1,~1) and (~2,~2), there is a minimizing/unc- tion between these points, and, i/ there are several such/unctions, that they are uni/ormly bounded. (The bounds will depend on ~1, ~h, ~ and ~2")

1 A sufficient condition for this is that limlz[-~r

F(x,y,

z) = + c~ uniformly for all x and y.

(9)

ARKIV FSR MATEMATIK. B d 6 n r 22

Then there is an absolutely m i n i m i z i n g / u n c t i o n / o r every minimization problem in the strip X I <~ x ~ X2, - ~ < y < ~ .

Proo/.

I n order to m a k e the proof more lucid, we shall divide it into several parts.

l) Consider the minimization problem between

(Xl,

Yl) and (x2, Y2). L e t ~ be the class of minimizing functions, which, b y assumption, is not empty.

P u t

[

u(x) ~ inf [(x) a n d

lv (x) = sup/(x).

f e T n

I t is clear t h a t

u(x)

and

v(x)

belong to ~ .

Consequently there are always a smallest minimizing function

u(x)

and a greatest minimizing function

v(x).

2) We shall introduce two notations in order to simplify the expressions later on.

L e t

p(x)

be a function on an interval I . Let [Xl,X2] be a sub-interval of I and consider the minimization problem between (Xl,

p(xl) )

and

(x2, p(x2)).

L e t

u(x)

be the smallest and

v(x)

the greatest minimizing function, as above. We shall agree to say t h a t

p(x)

has the p r o p e r t y A on I if

p(x)>~u(x)

on Ix1, xe]/or

every

sub-interval [xi, x2] of I and t h a t

p(x)

has the p r o p e r t y B on I if

p(x) <~v(x)

on

[xl, xeJ/or every

sub-interval Ix1, x2] of I . (Compare subharmonic and superharmonic functions.)

3) Consider our minimization problem on an interval I with some b o u n d a r y values. I t is obvious t h a t the greatest minimizing function

v(x)

is not less t h a n the

greatest

minimizing function

Vi(X )

between a n y two points (tl,

v(ti) )

and

(t2, v(t2) ).

Similarly, the smallest minimizing function

u(x)

is not greater t h a n the

smallest

minimizing function

u1(x )

between a n y two points

(tl, u(tl) )

and

(t2, u(t2) ).

4) We shall now construct an a.s. minimal.

From now on, we consider a /ixed minimization problem, namely between

(Xl, y~) and (x2, Y2). L e t G be the class of those minimizing functions which have the p r o p e r t y A on

[xl,x2]. G

is not empty, since the greatest minimizing function belongs to G.

We introduce the/unction

h(x)

= inf

g(x)

geG

and we assert that h(x) is an a.s. minimal.

I t is clear t h a t

h(x)

is a minimizing function.

5) Now let us prove t h a t

h(x)

has the properties A and B.

I) Choose an a r b i t r a r y interval it1, t2] a n d let

u(x)

be the smallest minimizing func- tion between

(tl, h(tl) )

a n d

(t2, h(t2) ).

L e t

g(x)CG.

We assert t h a t

g(x)>~u(x)

on t 1 ~<x ~ t 2. I f this were not true, then there would be an interval s 1 ~<x ~<s 2 such t h a t U(81)=g(81) , U(S2)=g(82) and

u(x)>g(x)

for

81<x<82.

Let

ul(x )

be the smallest minimizing function between (Sl,

u(sl) )

and

(se, u(s2) ).

I t follows from the definition of G t h a t

g(x)~>Ul(X

). B u t we also have

u ( x ) ~ u l ( x ).

(See p a r t 3 of the proof.) This gives

g(x)>~ul(x ) ~ u ( x )

and we have a contradiction.

Thus

u(x) <~g(x),

and the inequality

u(x) <~h(x)

follows from the definition of

h(x).

This proves t h a t

h(x)

has the p r o p e r t y A.

I I ) Assume t h a t

h(x)

has not the p r o p e r t y B. This means t h a t there is an interval It1, t2] such t h a t the corresponding greatest minimizing function

v(x)

does not satisfy 417

(10)

c . AROr~SSOrr Minimization problems. II

t h e inequality

v(x)>~h(x).

I t follows f r o m p a r t 3 of the proof t h a t we m a y assume t h a t

h(x)>v(x)

for t I < X < t 2. L e t us f o r m the function

h(x) for

x<~q, p(x)=]v(x)

for

tl<x<t2,

[h(x) for

x ~ t 2.

I f we can prove t h a t

p(x)E G,

t h e n this will contradict the definition of

h(x),

since we h a v e

p(x) < h(x)

on (tl, t2). I t is clear t h a t

p(x)

is a minimizing function on [xl, x~], a n d it remains to prove t h a t

p(x)

has t h e p r o p e r t y A. So let us consider a n interval Is1, s2] a n d let the corresponding smallest minimizing function be

ul(x ).

W e h a v e

ul(sl)=p(sl)<h(sO

a n d

ul(s2)=p(%)<~h(% ).

Since

h(x)

has t h e p r o p e r t y

A,

this

implies t h a t

h(x)~>Ul(X

). If p ( ~ ) < u l ( ~ ) for some ~E(Sl, S2), t h e n we m u s t h a v e

p(~)=v(~)<ul(~ ).

Consequently there m u s t be an interval [rl, r2] such t h a t

v(rl)

= u l ( r 0 ,

v(r2) -ul(r2)

a n d

v(x) < ul(x )

for r 1 < x < r 2. L e t

u2(x )

a n d

v2(x )

corre- spond to this interval a n d these b o u n d a r y values in the usual manner. T h e n we h a v e

udx)

<u2(x) <v2(x) < v(x).

B u t this contradicts t h e inequality

v(x)<ul(x

). This proves t h a t

h(x)

has the pro- p e r t y B.

6) I t is obvious t h a t

h(x)

satisfies the conditions I a n d I I which, as we h a v e seen, g u a r a n t e e t h a t t h e assertions in T h e o r e m 9' are true. Thus

h(x)E C 1

a n d the differential equation

dE(x, h(x), h'(x))

dx 9 Fz(x, h(x), h'(x)) = 0

is satisfied in t h e sense described there.

7) W e are n o w in a position to prove t h a t

h(x)

is an a.s. minimal. Consider an a r b i t r a r y interval t 1 < x < t 2. L e t M 0 be t h e m i n i m u m value of the functional, as usual, a n d p u t

M = m a x

F(x, h(x), h'(x)).

t x ~ x ~ t ~

W e w a n t to prove t h a t M = M 0.

I f

F(t 1,

h(tl), h'(t 0) = M, t h e n t h e result follows at once, since

h(x)

has the properties A a n d B, a n d t h e same is true for t 2. So let us assume t h a t

F(t 1,

h(tl),

h'(tl))<M

a n d t h a t

F(t~, h(t~), h'(t2))<M.

I t follows f r o m p a r t 6 t h a t there is a n u m b e r

2, tl<~<t2,

such t h a t h'(~)=r h(~)) a n d F(~, h(~), h'(~)) = M . L e t us assume t h a t M > M o

This means t h a t there is no minimizing function co(x) for which 0)(2)=h(~). (I.e.

(2, h(~)) CE(Mo). )

L e t K be the class of those minimizing functions a)(x), for which 0)(2)<h(~). (K is n o t e m p t y , see p a r t 5.) P u t k ( x ) = s u p ~ K co(x). I t is clear t h a t k ( x ) E K a n d t h a t k(~) < h(~).

P u t

/ 81 = sup {x ix < ~,

h(x)

= k(x)}

a n d )

[ s2 = inf {x ] x > ~,

h(x) = k(x)}.

(11)

ARKIV FOR MATEMATIK. B d 6 n r 22 W e introduce the function

h(x) for

x < s 1, q(x)= jk(x)

for

s l < x < s 2,

[h(x) for x ~ s 2.

If we can prove t h a t

q(x)E G,

t h e n this will c o n t r a d i c t the definition of h(x), since

q(x)

<h(x) on (s 1, s2). (Compare p a r t 4.)

So all we have to prove is that q(x) has the property A.

Consider t h e n a n a r b i t r a r y interval [rl, r2] a n d let

u(x)

be t h e smallest minimizing f u n c t i o n between

(rl, q(rl) )

a n d

(r2, q(r2) ).

W e h a v e

u(rl)=q(ri)<~h(rl)

a n d

u(r2)

=q(r2)

<~h(r2).

Since

h(x)

has the p r o p e r t y A, it follows easily t h a t

h(x) >~u(x)

for r~ ~< x ~< r e.

So if we assume t h a t

u(xo)>q(x0)

for some x0, t h e n we m u s t h a v e

s 1 <xo<s 2

a n d

q(xo) =k(x0).

Consequently there m u s t be a n interval (~h, ~2) such t h a t s 1~<~1 <~2 ~<s2, u(~h) = k(~l), u(~2) = k(~2) a n d

u(x) > k(x)

for ~1 < x <V2.

If

H(k; ~ , ~2) <~H(u; ~ , ~2),

t h e n we get a contradiction to the definition of

u(x).

Suppose t h e n t h a t L e t us form the function

H(k; ~1,

~ 2 ) > H ( u ; ~1,

~2)"

[

~(X) for Z ~ l ,

kl(x ) = j u ( x )

for ~ 1 < x ~ 2 ,

[k(x) for x ~ 2 .

Clearly, kl(X )

is a minimizing function between (tl,

h(tl) )

a n d (t~,

h(t2) ).

F u r t h e r , we h a v e kl(~)~<h(~ ) (for, as we m e n t i o n e d above,

u(x)~h(x)

for all x, where

u(x)

is defined). H e n c e kl(x ) EK. :But this contradicts the definition of k(x). Hence we get a contradiction in a n y case.

This completes the proof of T h e o r e m 2.1.

3. The m e t h o d of a p p r o x i m a t i n g a m a x i m u m b y a sequence of integral m e a n values is well known. I n our case it means t h a t we should consider t h e functional

H(/)

as the limit of the sequence of functionals

H n ( l ) = 1 ,

[F(x,/(x),f(x))]ndx] 11~,

n = 1 , 2 , 3 , . . . .

W e have used this a p p r o a c h in [1] to derive t h e differential e q u a t i o n

(dF/dx). F z = O.

I t can also be used to give a different existence proof for a.s. minimals. This is v e r y n a t u r a l since a minimizing function in the calculus of variations a u t o m a t i c a l l y is minimizing on e v e r y sub-interval. However, we shall n o t c a r r y t h r o u g h this proof since it is more complicated t h a n the one already given, a n d since it requires stronger conditions on

F(x, y, z).

A n o t h e r result which can be p r o v e d with this "integral m e t h o d " is the following:

if/o(X) is t h e only a.s. minimal for

H(/)

a n d if/~(x) minimizes

Hn(/)

for n = 1, 2, 3, ..., t h e n limn_~ ~/~(x) =/o(X) uniformly.

419

(12)

c. ARONSSON, Minimization problems. II

Chapter 3. Further examination of absolutely minimizing functions.

Uniqueness questions

For reasons of simplicity, this chapter will treat only the case

(o(x,y)=-O.

N o doubt, the results can be modified for the general case.

1. The following theorem gives us further information on the structure of absolutely minimizing functions:

Theorem 3.1.

Assume that F(x, y, z) satis/ies these conditions:

F(x, y, z) is an analytic/unction o/ x, y and z in a complex neighbourhood o/the set o/ values

{

X l ~ X ~ X 2

y, z real;

2 A and 3.

Assume/urther that/(x) is an absolutely minimizing/unction on the compact interval xl ~ x ~ x 2.

Then:

a)

/(x) EC 1 on

[xl, x2]

(already proved);

b)

the set {x I F~(x, /(x), /'(x)) # 0 } consists o / a f i n i t e number o/intervals (to be proved now);

c)

the di//erential equation

dF(x, /(x), /'(x)) Fz(x, /(x), /'(x)) = 0 dx

is satis/ied in the sense that F(x, /(x), /'(x)) is constant on each o/these intervals (al- ready proved).

Proo/.

I n order to facilitate the further references, we divide the proof into several parts.

1) Assume t h a t

] ' ( x ) # 0

for

p < x < q

and t h a t / ' ( p )

=/'(q)=0.

Fx(p,/(p), o) < o,

Then /

[ Fx(q,/(q), O) >10.

To see this, we recall t h a t / ( x )

EC 2

on

(p,q)

and t h a t

F(x, /(x), /'(x))

is constant there (Theorem 8 in [1]). So we have

Fx(X, /(x), /'(x)) + F~(...) /'(x) + Fz(...)/"(x) = 0

for

p < x < q .

Clearly, there m u s t be points arbitrarily close to p, where

/'(x)

a n d

/"(x)

have the same sign. At such a point

Fz(...)/"(x)>0,

which gives

Fx(x, l(x), l'(x)) + ~'y(...) /'(x)

< 0 . 420

(13)

ARKIV FOR MATEMATIK. B d 6 n r 2 2 Since l i m x - ~ v / ' ( x ) = O , t h i s p r o v e s t h e first i n e q u a l i t y , a n d t h e s e c o n d is p r o v e d s i m i l a r l y (there a r e p o i n t s a r b i t r a r i l y close t o q w h e r e

Fz(...)/"(x ) <0,

etc.).

N o w s u p p o s e t h a t t h e r e is a n u m b e r

t>q

such t h a t / ' ( t )

4=0.

T h e n i t follows f r o m t h e a b o v e r e s u l t t h a t we can define

r=inf{xlx>~q,

F~(z,/(z), 0)=0},

a n d i t follows t h a t

q<r<t.

I f

q<r,

t h e n we o b v i o u s l y h a v e

/'(x)=O

for

q~x<~r

( a n d if q = r , t h e n t h i s is t r i v i a l l y true).

I f t h e r e is a n u m b e r

u < p

such t h a t

]'(u)4=0,

t h e n we define s = s u p {x; x < p , F ~ ( x , / ( x ) , 0) = 0 } . W e h a v e / ' ( x ) = 0 for

s ~ x ~ p .

F i n a l l y , we o b s e r v e t h a t

{ Fx(x,/(x),O)~O

for

s<~x~p, Fx(x,/(x),

0)~>0 for

q<~x<~r.

2) W e shall give a n i n d i r e c t p r o o f of t h e t h e o r e m . So we a s s u m e t h a t t h e set {x[ x 1 < x < x2,

/'(x)

4 0 } is t h e u n i o n of a n i n f i n i t e n u m b e r of d i s j o i n t o p e n i n t e r - vals. These i n t e r v a l s m u s t h a v e a l i m i t - p o i n t a n d we m a y a s s u m e t h a t i t is

x=O.

W e m a y also a s s u m e t h a t / ( 0 ) = 0 a n d t h a t t h e r e a r e i n f i n i t e l y m a n y such i n t e r v a l s i n e v e r y i n t e r v a l 0 < x < e. C l e a r l y / ' ( 0 ) = 0 a n d Fx(0, 0, 0) = 0.

T h e f u n c t i o n

Fx(x,

y, 0) will b e i m p o r t a n t for t h e r e s t of t h e proof. I t m a y be i d e n t i c a l l y zero or not. T h i s gives us t w o cases a n d we shall s t a r t w i t h t h e s i m p l e s t of t h e m .

3) A s s u m e t h a t Fx(x, y, 0) ~ 0. T h i s m e a n s t h a t we h a v e

F(x, y, O) = F(O, y, O) =of(y),

w h e r e ~c(y) is a n a l y t i c in a (complex) n e i g h b o u r h o o d of y = 0 .

A) ~c(y)--- c o n s t a n t , i.e.

F(x,

y, 0) --- c o n s t a n t . Consider a n i n t e r v a l

(p, q)

as in 1). P u t

t=89 +q).

T h e n we h a v e

F(t, /(t), /'(t)) > F(t,/(t), 0) =

F(p, tip),

0),

w h i c h gives a c o n t r a d i c t i o n .

B) ~c(y) ~ c o n s t a n t .

I t follows f r o m T h e o r e m 10 in [1] 1 t h a t / ( x ) is m o n o t o n i c , for i n s t a n c e n o n - d e c r e a s - ing.

B u t ~ c ' ( y ) # 0 for 0 < y < ~. H e n c e ~c(y) is s t r i c t l y m o n o t o n i c for 0 ~ y ~ . Consider a n i n t e r v a l

(p,q)

such t h a t

O</(p)</(q)<~.

T h e n we m u s t h a v e

cf(/(p))=cf(/(q))

w h i c h gives a c o n t r a d i c t i o n .

1 I t is c l e a r t h a t t h e c o n d i t i o n o n F z i n T h e o r e m 10 n e e d o n l y b e a s s u m e d t o h o l d for z = 0.

I n f a c t , t h e c o n d i t i o n of T h e o r e m 1.1 i n t h i s p a p e r is s u f f i c i e n t if i t h o l d s f o r a l l y.

4 2 1

(14)

G. AROr~SSOr~, Minimization problems. II

4) ~qow let us assume t h a t

F~(x,

y, 0) ~ 0. Clearly

F~(x, y,

0) is a n analytic function a n d we can a p p l y Weierstrass' p r e p a r a t i o n t h e o r e m (see [2], p. 89). I n our case it says t h a t

F~(x, y, O) =x~o~(x, y)W'(x, y)

in a n e i g h b o u r h o o d U of the origin. Here # is an integer ~>0;

~o(x,y)

is analytic in U a n d # 0 ; ~F(x, y) is

either =- 1 or

of t h e f o r m

vt~(x ' y) =ym + A~(x)y,~-i + A2(x)ym-2 + ... + A,~(x),

where t h e functions

Ak(x)

are analytic in a (sufficiently small) n e i g h b o u r h o o d of

x=O

a n d A k ( 0 ) = 0 for all k.

W e k n o w f r o m 2) t h a t

F:~(x,

y, 0) - 0 at t h e points (r,/(r)) a n d

(s, ](s)).

Therefore we can exclude t h e case ~F ~= 1 and, instead, we inquire a b o u t the zeros of

uz.(x,y ) =ym + A~(x)yr~-I + ... +Am(x).

L e t us, for the present, replace x a n d y b y the complex variables ~ a n d ~]. W e shall t r y to determine ~ as a function of $ f r o m t h e relation ~F(~,~) = 0 . W e are only in- terested in the solutions of this e q u a t i o n in a n e i g h b o u r h o o d of ~ = 9 = 0 .

Consider a complex n e i g h b o u r h o o d V of $ = 0 a n d c u t it along t h e negative real axis. T h e n every root of ~ can be defined as a regular function in V, a n d the roots

~k of ~F(~, ~]) = 0 m a y be written

= ~ C i~l/n~

for k = l , 2 , . . , m . ,~,o, j , k ~ ,

Concerning this expansion, see [3], p. 50; [4], pp. 98-103 a n d [5], Chapter X I I I . (The roots can be divided into cyclic systems, a n d the n u m b e r n can be chosen as t h e p r o d u c t of the n u m b e r of roots in each system.)

W e m a y assume t h a t ~l/n is real for ~ > 0, which gives the expressions

~ ~ - ~ J . k [ x / 1/n~i ) ~ k = 1, 2, . . . , m.

j = l

Clearly, we need only consider those series in which Cj.g are real for all ?'. (If there are no such series, t h e n there is n o t h i n g left to prove.) Therefore, let us suppose t h a t

~]k(x),

k = 1,2 ...

M,

are real for 0 < x < 0 , a n d t h a t ~k(x) are complex (not real) for k > M a n d 0 < x < 0 . W e k n o w f r o m t h e first p a r t of t h e proof t h a t

Fx(r,/(r),

0)= Fx(8,/(s), 0)=0.

H e n c e t h e points (r,/(r)) a n d

(s,/(s))

m u s t lie on the curves

y =~k(x)

if r a n d s are sufficiently close to x = 0.

(15)

ARKIV FOll MATEMATIK. B d 6 n r 22 5) W e shall n o w s t u d y in some d e t a i l t h e f u n c t i o n s

~k(x) -= ~ ~].k

( x l / n ) ~ = g k ( X l / n ) 9

j = l

H e r e t h e f u n c t i o n s

g~(z)

a r e a n a l y t i c for ]z[ < 8. F u r t h e r , e v e r y

g~(z)

is r e a l if z is real.

D i f f e r e n t i a t i o n gives

. _ _ - 1 - - 1

,(X)=gk(xl/n ) 1 "X n

for

O < x < 8 n.

n

P u t t =

x ~/n,

w h i c h g i v e s

~k (x) = g; (0 [ ~ = ~ ( t ) .

T h e f u n c t i o n ~ ( z ) is a n a l y t i c in 0 < I z] < 8 a n d t h e s i n g u l a r i t y a t z = 0 is e i t h e r re- m o v a b l e or a pole. W e h a v e

l i m ~ (x) = l i m F~(t),

x --.-~ + 0 t ' - - > + O

a n d t h i s l i m i t m u s t be real. I t can b e + o% - 0% f i n i t e b u t ~ 0 a n d 0. Since ]'(0) = 0 , we can e x c l u d e f r o m c o n s i d e r a t i o n t h o s e v a l u e s of/c for which we do

not

h a v e ~ ( 0 ) = 0 . T h e r e f o r e we c a n a s s u m e t h a t ~ ( z ) is a n a l y t i c for l zl < ~ .

A c o n s e q u e n c e of t h i s is t h a t t h e r e is a 8 1 > 0 such t h a t ~vk(z ) 4=0 for 0 < ]z] <81, unless ~vk(z)~0. F r o m t h e r e l a t i o n ~ ( x ) =~k(xl/~), we can infer a c o r r e s p o n d i n g r e s u l t for ~ ( x ) .

N o w let us c o m p a r e t h e f u n c t i o n s ~ ( x ) =gk(t) a n d ~ l ( x ) - g l ( t ) . T h e f u n c t i o n s

gk(z)

a n d

gz(z)

a r e a n a l y t i c a n d n o t i d e n t i c a l . H e n c e t h e r e m u s t e x i s t a 82 > 0 such t h a t

9k(z)#gl(z)

for 0 < l z ] <82.

T h i s gives a c o r r e s p o n d i n g r e s u l t for ~k(x) a n d ~z(x).

W e h a v e f o u n d t h a t ~/k(x)=gk(x

1/'~)

a n d ~ ( x ) = ( v k ( x l / ~ ) , w h e r e

gk(z)

a n d ~k(z) a r e a n a l y t i c for Iz[ < 8 . L e t us w r i t e

xl/n=t,

as a b o v e . This gives

~'(x, ~(x), o)= ~'(t ~, g~(t),

o ) = u ( t ) a n d

F(x, ~k(x), ~'k(x) )= F(t ~, gk(t), qJk(t) )=v(t).

Clearly, t h e f u n c t i o n s

u(z)

a n d

v(z)

are a n a l y t i c in a n e i g h b o u r h o o d of z = 0 . H e n c e t h e r e is a 8~ > 0 such t h a t e a c h of t h e f u n c t i o n s

F(x,

~k(x), 0) a n d

F(x, ~(x), ~'k(x))

is e i t h e r increasing, c o n s t a n t or d e c r e a s i n g on t h e whole i n t e r v a l 0 ~< x ~< 83.

W e h a v e e x c l u d e d t h o s e f u n c t i o n s

~k(x),

for w h i c h we d i d n o t h a v e ~]~(0) = 0 . A f t e r a r e n u m b e r i n g , we h a v e ~k(x), k = 1, 2 . . . P , left.

I f we collect o u r results, t h e n we see t h a t t h e r e e x i s t n u m b e r s ~ > 0 a n d % such t h a t : a)

~k(X):4:~l(X)

if 0 < X ~ : r a n d

lc:4:l.

This m e a n s t h a t one of t h e f u n c t i o n s ~k(x) is s m a l l e s t a n d one is g r e a t e s t on t h e whole i n t e r v a l 0 < x ~< ~.

b) ~]k(x) C C 1 on 0 ~<x ~ cr a n d ~k(0) =~/~(0) = 0 . I f ~k(x) ~: 0, t h e n ~ ( x ) 4=0 for 0 < x ~< ~.

c) I f we h a v e p , q, r a n d s as in 1) a n d 0 < s < r ~ < c ~ , t h e n

](r)=~k(r )

for s o m e

k<~p

a n d

](s)=~(s)

for s o m e k 1 ~<P.

423

(16)

c. xRo~ssorr

Minimization problems. I I

d) E v e r y function

F(x, ~k(x),

0) is either c o n s t a n t or strictly m o n o t o n i c on 0~<x~<~. The same is true for

F(x,

~k(x), ~ ( x ) ) (1

<~k<~P).

6) W e are interested only in those functions ~k(x) for which one of t h e relations u n d e r c) above really occurs on t h e interval 0 < x ~< ~. I f we exclude t h e others, t h e n we get (after a renumbering) t h e functions ~k(x), k = 1, 2 . . . . , N, left. This does n o t affect t h e validity of t h e s u m m a r y a)-d).

I f N = 1 a n d ~ l ( x ) ~ 0 , t h e n we get a contradiction at once a n d there is n o t h i n g left t o prove.

I f this is n o t t h e case, t h e n t h e greatest of t h e functions ~ ( x ) is > 0 (for 0 < x ~< r162 o r t h e smallest is < 0.

L e t us choose t h e first case a n d let ~N(X) be the function in question. T h e n

~N(X) > 0

a n d ~]N(X) > 0 on 0<x~<r162

I t follows from our choice of

~N(X)

t h a t there is an interval (P0, %) such t h a t / ( % )

=

~N(ro).

W e have t h u s

/(qo)--/(ro)=~N(ro);

/'(qo) = 0 a n d ~N(X) > 0 .

H e n c e

/ ( x ) > ~ ( x )

for q0 - ~ < x < % . B u t we also h a v e / ( P o ) =/(So)<~N(So) (owing t o our choice of ~ ( x ) ) a n d

/'(po)-O.

H e n c e

/ ( x ) < ~ ( x )

for

p o < x < p o §

Conse- q u e n t l y there m u s t be a ~, p 0 < ~ < q 0 , such t h a t / ( ~ ) = ~ ( ~ ) a n d / ' ( ~ ) > ~ N ( ~ ) > 0 .

N o w assume first t h a t

F(x,

~?N(X), 0) is c o n s t a n t or decreasing. T h e n we have

F(~, ~N(~), O) >~ F(r o, ~]N(rO), O) >~ F(q o, ](qo), 0).

(See t h e first p a r t of the proof.) This gives

F(8, /(8), /'(8))

> F(q0,/(qo),

0),

which is a contradiction.

Assume t h e n t h a t

F(x, ~N(X),

0) is increasing. A n obvious consequence of this is t h a t

F(x, ~N(X), ~N(X))

is increasing. L e t us n o w consider t h e minimization problem between t h e points x l = y l = 0 a n d x2=~, Y2=/(~)=~N(~). Since

/(x)

is an absolute minimal, we h a v e

M o = H ( / ) >~F(~,/(~), 1'(~)).

:But

F(x, ~TN, ~7"N)

is increasing, which gives

M o <~ H(~tN ) = F(~, ~IN(~), ~'N(~)) <~ H(/) = M o.

T h u s

H(~tN ) = M o

a n d

~N(X)

is also a minimizing function.

Choose a function r C 1 on 0 ~<x ~<~ such t h a t ~ ( 0 ) = r a n d r 0. F o r m t h e function

g(x)-~N(x)+),~b(x),

where A > 0 is a parameter. W e h a v e

.F(x, g(x), g'(x) ) = F(x, ~N(x), V]'N(X) ) § ~.(a(x)r d-b(x)r ) § R(x, 2.).

H e r e

a(x) = Fy(x, ~N(x), ~7'N(X))

a n d

b(x) = F~(x,

~N(x), ~v(x)) a n d [

R(x, 4) 1 <~ C~ 2

where C is i n d e p e n d e n t of x a n d 2, if 0 < ~ < 1.

Now, recalling the fact t h a t

F(x, ~TN, ~'N)

takes its m a x i m u m o n l y at x = ~ a n d t h a t F~(~, ~N(~), ~TN(~)) > 0 , it is easy to verify t h a t

H(g)

< H ( ~ ) , if A is small enough.

This gives

H(g) < M o

which is a contradiction. This completes the proof.

(17)

ARKIV FOR MATEMATIK. B d 6 n r 2 2 2. This section will t r e a t b r i e f l y t h e q u e s t i o n of u n i q u e n e s s for a.s. m i n i m a l s . I t will be s h o w n b y e x a m p l e s t h a t t h e r e m a y be s e v e r a l a.s. m i n i m a l s for a g i v e n mini- m i z a t i o n p r o b l e m . A few sufficient c o n d i t i o n s for u n i q u e n e s s will also be given.

Choose F(x, y, z) = (1 + x ~ + y2)2 § z2, 2

x 1 = - 2 , yl=O, x 2 = 2 a n d y ~ = 0 .

I t follows f r o m T h e o r e m 2.1 t h a t t h e r e is a n a.s. m i n i m a l / ( x ) . Clearly, g(x) = - / ( - x ) is also a n a.s. m i n i m a l . Now, if t h e r e is a u n i q u e a.s. m i n i m a l , t h e n we g e t - ] ( - x ) ](x), i.e. /(0) = 0. C o n s e q u e n t l y M o >~ F ( 0 , 0, 0) = 2.

P u t h ( x ) = 1 2 - x for x~>0,

[2+

x for x ~ 0 . C l e a r l y H(h) = 2 / 9 § 1 < 2.

T h e c o n t r a d i c t i o n shows t h a t t h e r e is no u n i q u e a.s. m i n i m a l . (This s i t u a t i o n c a n b e d e s c r i b e d thus: e v e r y m i n i m i z i n g f u n c t i o n m u s t e v a d e t h e m a x i m u m a t x = y = 0 , w h i c h , owing t o t h e s y m m e t r y , l e a d s t o n o n - u n i q u e n e s s . )

P u t F(x, y, z ) = y 2 + z 2 a n d let /o(X) be d e f i n e d as in E x a m p l e 3 in [1]. W e k n o w t h a t all f u n c t i o n s p/o(x+q)(p, q a r e c o n s t a n t s ) a r e a.s. m i n i m a l s . I t is e v i d e n t t h a t t h e r e a r e i n f i n i t e l y m a n y a.s. m i n i m a l s if Y2 = - Y l 44=0 a n d x 2 - x 1 >~r.

I t follows f r o m t h i s e x a m p l e t h a t t h e r e m a y b e s e v e r a l a.s. m i n i m a l s e v e n if 2'(x, y, z) is c o n v e x in y a n d z. I n t h e calculus of v a r i a t i o n s , t h i s c o n d i t i o n l e a d s t o u n i q u e n e s s (for all b o u n d a r y values). I t follows f r o m t h i s e x a m p l e t h a t t h e c o n d i t i o n (Yl - C) (Y2 - C) ~> 0 in T h e o r e m 3.3 c a n n o t b e o m i t t e d .

These e x a m p l e s s h o w t h a t e x t r a c o n d i t i o n s on F(x, y, z) o r on t h e b o u n d a r y v a l u e s m u s t be a d d e d to o u r u s u a l ones in o r d e r to secure uniqueness. Two w a y s to do t h i s a r e s h o w n b y t h e following t h e o r e m s :

T h e o r e m 3.2. Assume that F(x,y,z) satisfies the conditions o/ Theorem 2.1, with o)(x, y ) ~ 0 . Then, as we know ]rom that theorem, there is at least one a.s. m i n i m a l / o r

every choice o / ( x D Yl) and (x2, y~).

Assume now that

~Y(x, y, O)

0 for all x and y.

~x

Then there is a V•IQVE a.s. minimal/or every choice o/(xl, Yl) and (x2, Y2).

Proo/. W e a s s u m e t h a t Fx(x,y,O ) > 0 . W e m a y also a s s u m e t h a t Yl <Y2 (the case yl>y2 is a n a l o g o u s a n d t h e case Yl=Y2 is t r i v i a l ) . L e t /(x) b e a n a.s. m i n i m a l . I f ]'(x) > 0 for x 1 ~ x <~x2, t h e n t h e a s s e r t i o n follows f r o m t h e T h e o r e m s 6 a n d 9 in [1].

I f / ' ( x ) =0 for some x, t h e n i t follows f r o m p a r t 1 of t h e p r o o f of T h e o r e m 3.1 t h a t t h e r e is a n u m b e r ~, x 1 < ~ ~x2, such t h a t

> 0 for xl<~x<~, /'(x) is = 0 for ~ < x ~ < x ~ .

425

(18)

a. ARO~SSO~, Minimization problems. I I

I f g(x) is a different a.s. minimal, then the same reasoning holds for g(x). Let ~' correspond to g(x). Assume t h a t ~' <~. This gives g'(xl) >]'(xl) and

B u t we also have

M1: F(Xl,

Yl, /' (xl) ) < F(xl, Yl, g' (x) ) = M e.

M 1 = F ( ~ , Y2, 0) > F(~', Ye, 0) = M e.

The contradiction proves the theorem.

Theorem 3.3. Assume that F(x,y,z) satis/ies these conditions:

F(x,y,z) is analytic, as in Theorem 3.1;

2 A and 3;

the condition concerning existence and boundedness o/minimizing/unctions, which was given in Theorem 2.1;

t > : i / y > C '

~F(x,y,Z) is = i/ y = C ,

@ [ < 0 i/ y < C , (C is a constant);

F(x, ~Yl + ( 1 - 2 ) y~, 4z 1 + (1-4)z2) ~<~F(x, y~, Zl) + ( 1 - 4 ) F(x, y~, z~)

/or all x, Yl, Y2, zl and z e and/or all 4 such that 0 < 4 < 1 , equality holds i/ and only i/

Yl =Y~ and z I =z2.

We consider the minimization problem between (x 1, Yl) and (x 2, Ye). We assume that the boundary values satis/y the inequality

(Yl - C) (Ye - C) >~ O.

Then there is a VNIQV]~ a.s. minimal between (x 1, Yl) and (xe, Ye).

The proof is omitted, since it is rather laborious and since the result is not used in this paper.

3. I n the previous section, we discussed the uniqueness question for a.s. minimals.

We shall now briefly consider the same question for minimizing functions. The following theorem shows t h a t those functions F(x,y,z), for which every minimization problem has a unique solution, constitute a v e r y "small" class.

Theorem 3.4. Let F(x, y, z) satis/y the/ollowing conditions/or X 1 <~x <~X2 and all y,z:

F(x, y, z) E Ce;

2 A and 3;

the condition concerning existence and boundedness o/ minimizing /unctions which was given in Theorem 2.1.

We consider the minimization problem between (x~, y)and (xe, Y2) (X1 <~xl <x2 <<-X2) 9 Then there is a VNTQV~ minimizing /unction /or every choice o/ xl, Yl, x2 and Y2 i/ and only i/

426

(19)

ARKIV FOR MATEMATIK. B d 6 n r 2 2

_F(x, y, O) =-constant.

Proof.

1) S u p p o s e t h a t

F(x,

y, 0) - c o n s t a n t . Consider t h e m i n i m i z a t i o n p r o b l e m b e t w e e n (xl, Yl) a n d (x2, Y2). W e k n o w t h a t t h e r e is a n a.s. m i n i m a l / ( x ) a n d we k n o w t h a t / ( x )

E C 1.

If ] ' ( x ) 4 0

for x 1 ~<x ~<x~ t h e n i t follows f r o m T h e o r e m 8 a n d T h e o r e m 6 in [1] t h a t

](x)

is a u n i q u e m i n i m i z i n g f u n c t i o n .

I f t h e r e is a n ~ such t h a t / ' ( r 1 6 2 = 0 , t h e n

] ' ( x ) - 0 .

I n o r d e r to see t h a t , s u p p o s e t h a t

/'(fl) # 0

for s o m e fi > ~. :Put

= sup {x Ix < ~ , / ' ( x ) = o}.

T h e n we g e t

F(/~, / @ , / ' @ ) > P ( ~ , 1 @ , 0) = F ( ~ , / ( r ) , 0).

B u t this c o n t r a d i c t s t h e r e l a t i o n

F ( x , / , / ' )

= c o n s t a n t w h i c h h o l d s o n e v e r y i n t e r v a l w h e r e

]'(x) #0.

So we h a v e

](x)-yl(=y2).

I f

g(x)

is a n a d m i s s i b l e f u n c t i o n a n d g ' ( ~ ) # 0 for s o m e

~, t h e n we g e t

H(g)

>~ F ( $ , g(~), g'($)) > F(~, g(~), 0) = H ( / ) , a n d we h a v e p r o v e d one h a l f of t h e t h e o r e m .

2) S u p p o s e n o w t h a t t h e r e is a u n i q u e m i n i m i z i n g f u n c t i o n for e v e r y choice of xl, x2, Yl a n d Y2.

I f F=(Xl, Yl, 0) =4=0, t h e n we choose Y2 =Yl a n d x 2 such t h a t Fz(X , Yl, 0) # 0 for

xl<~x<<.x ~.

Clearly,

/ ( x ) - Y l

is a m i n i m i z i n g f u n c t i o n , b u t n o t t h e o n l y one. T h i s p r o v e s t h a t

Fx(x , y, O) - O.

I f

Fy(Xl,

Yx, 0) # 0 , t h e n we choose

Yz=Yl

a n d x 2 such t h a t

Fy(x,

Yl, 0) # 0 for x 1 ~<x ~<x z. W e shall p r o v e t h a t / ( x )

=Yl

is n o t t h e o n l y m i n i m i z i n g f u n c t i o n .

I t is no r e s t r i c t i o n to a s s u m e t h a t x 1 =Yl = 0 a n d Fy(0, 0, 0 ) < 0. Consider t h e func- t i o n y = ax2 w h e r e ~ > 0 is a c o n s t a n t . T a y l o r s f o r m u l a gives

F ( x , ~x 2, 2~x) =

F(x, O, O) + ax~F~(x, O, O) + l(~2xaF~(x, O. ax2, O.

2ax) + 2. ax~ .2~XF~z(...) + 4~x~F~z(...)).

I f we consider a s u i t a b l e n e i g h b o u r h o o d of t h e o r i g i n a n d a s s u m e t h a t 0 < ~ < 1, t h e n F ~ ( x , 0 , 0 ) < - K < 0 , a n d t h e m o d u l u s of t h e e x p r e s s i o n in b r a c k e t s is n o t g r e a t e r t h a n K 1 9 ~2. x 2, w h e r e K a n d K 1 a r e i n d e p e n d e n t of x a n d ~.

This m e a n s t h a t

F(x, ax e,

2ax) ~<

F(x,

0, 0) - K . ~x 2 + K 1 ~2x2 =

F(x,

0, 0) -

ax2(K -

~K1).

I f we choose ~ so s m a l l t h a t K - ~ K 1 > 0, t h e n we h a v e

F(x, ax2,

2~r ~<

F(x,

0, 0) for 0 ~ x < ~ , for s o m e 8 > 0 .

A s i m i l a r c o n s t r u c t i o n c a n be c a r r i e d t h r o u g h for @2, Y2), a n d t h e r e s t of t h e p r o o f is obvious.

Therefore, if F ( x , y, 0) is n o t c o n s t a n t , t h e n we m u s t i m p o s e c o n d i t i o n s on t h e b o u n d a r y v a l u e s in o r d e r t o secure u n i q u e n e s s . T h e following t h e o r e m i l l u s t r a t e s t h i s p o s s i b i l i t y .

427

(20)

G. ARO~SSOrr Minimization problems. I I

T h e o r e m 3.5. Assume that F(x, y, z) satisfies these conditions:

F(x, y, z) E C1;

2 A and 3;

the condition concerning existence and boundedness o/ minimizing /unctions which was given in Theorem 2.1;

either ~F(x, y, O) >~ 0 / o r all (x, y) or < 0 / o r all (x, y).

~x

Let us denote the admissible linear/unction by l(x). Finally, we assume that m i n F(x, l(x), l'(x)) > m a x F(x, y, 0),

X I ~ X ~ X ~ ( X , y ) e K

where K is the set I xl ~ x <~ x 2

[

Yl <~ Y <~ Y2 (Yl >t Y >1 Y~)"

Then there is a unique minimizing ]unction/(x). Moreover, ]'(x) # 0 /or x 1 ~x<~x2.

Proo/. W e k n o w t h a t t h e r e is a n a.s. m i n i m a l / ( x ) EC 1. I f / ' ( x ) = 0 for some x, t h e n i t follows f r o m t h e T h e o r e m s 9 a n d 10 (with a t r i v i a l change) in [1] t h a t

M 0 ~ m a x F(x, y, 0).

(X,y) E K

B u t i t was p r o v e d in L e m m a 5 in [1] t h a t M0>~minxl<x<x ~ F(x, l(x), l'(x)). So t h e as- s u m p t i o n t h a t ]'(x)~0 for s o m e x l e a d s t o a c o n t r a d i c t i o n . F i n a l l y , i t follows f r o m T h e o r e m 6 in [1] t h a t ](x) is a u n i q u e m i n i m i z i n g f u n c t i o n .

Remark. I t is e a s y t o f i n d n e w r e s u l t s of t h e s a m e t y p e b y using n e w e s t i m a t e s . I t s h o u l d be m e n t i o n e d h e r e t h a t t h e i n e q u a l i t y

M 0~> inf F(x,g(x),g'(x))

x1~x<~x2

h o l d s for e v e r y a d m i s s i b l e m o n o t o n i c f u n c t i o n g(x)EC 1 ( c o m p a r e L e m m a 5 in [1]).

This gives

M 0 >~ sup [ inf F(x, g(x), g'(x))],

g(x) xl<~x~x~

w h e r e t h e s u p r e m u m is t a k e n o v e r all such f u n c t i o n s .

I t is also clear t h a t t h e c o n d i t i o n on F x in t h e t h e o r e m c a n b e r e p l a c e d b y t h ~ c o n d i t i o n in T h e o r e m 1.1 (in t h i s case a s s u m e d t o h o l d for all y).

Chapter 4. Comments and examples

I n t h i s c h a p t e r , we shall i l l u s t r a t e some of t h e t h e o r e m s b y m e a n s of e x a m p l e s . Example 1. S u p p o s e t h a t

F(x, y, z) = G(x, y) + A y 2 + By + z 2,

w h e r e G(x,y) is c o n t i n u o u s a n d b o u n d e d f r o m b e l o w a n d A , B are c o n s t a n t s .

(21)

ARKIV FOR MATEMATIK, B d 6 n r 2 2 Is there a minimizing function? W e shall a p p l y T h e o r e m 1.2'. F o r m a l calculation gives

l O(x, y , M ) = § (M - G ( x , y) - A y e - B y ) ~/2, 9 (x, y, M) = - ( M - G(x, y) - A y 2 - By) 1/2.

I f :r is defined for y

~Yl,

t h e n we see f r o m the expression for O(x,y, M) t h a t a(y,M) =O(y), when y--> + ~ . H e n c e t h e integral 11 in T h e o r e m 1.2' c a n n o t exist finite. I t follows in the same w a y t h a t none of t h e integrals Ie, /3 a n d 14 can exist finite.

So we can conclude t h a t E ( M ) is b o u n d e d for all M. Consequently there is a mini- mizing f u n c t i o n for e v e r y choice of xl, x2, Yl a n d Y2-

Example 2. W e h a v e n o t studied t h e properties of minimizing functions in general, we h a v e o n l y studied the special cases of a.s. minimals a n d unique minimizing func- tions. The reason for this is clear f r o m t h e following example:

Choose F(x, y, y') = x + y,2, xi = Yl = 0, x 2 = 1 a n d Y2 = 0. This gives M o >~ F(x~, y~, O) = 1, a n d if g(x)~-0, t h e n H(g)= 1.

Thus we h a v e M 0 = 1. I t follows t h a t i f / ( x ) is an admissible function, t h e n it is a minimizing f u n c t i o n if a n d only if I/'(x) l ~ V i - x a.e.

Clearly, the fact t h a t / ( x ) is a minimizing function implies v e r y little a b o u t / ( x ) . This m o t i v a t e s the i n t r o d u c t i o n of a.s. minimals.

On the other hand, suppose t h a t we h a v e a minimizing function g(x) which belongs to C 1. T h e n the local v a r i a t i o n m e t h o d , which was used in the end of t h e proof of T h e o r e m 3.1, can be applied to g(x). This m e t h o d works also in t h e general case of variable ~o(x,y). F o r instance, it can be used to derive t h e following result: if

Fz(X, g(x), g'(x)).o

for all x, t h e n F(x, g(x), g'(x)) = M o for all x. (This is m e n t i o n e d in [6] with a s k e t c h of a proof.) This result leads us once again to the differential equation (dF/dx)" Fz = 0 for a.s. minimals.

Example 3. W e h a v e p r o v e d in Chapter 2 t h a t if F(x, y, z) satisfies certain condi- tions, t h e n there is an a.s. minimal for e v e r y choice of the b o u n d a r y values, a n d we h a v e also p r o v e d t h a t every a.s. minimal belongs to C 1.

Consequently, there is a minimizing/unction in C 1.

However, there need n o t exist a minimizing function in C 2, as can be seen f r o m the following example:

P u t F(x, y, y') =y,2_cos2x, xl =Yl =0, x 2 =n7~ a n d Y2 = 2 n . W e assert t h a t

/(x)= [cos t l dt

is the only minimizing function. Clearly,/'(x) >~0 a n d F(x, /(x), /'(x)) - 0 . If H(g) <~0, t h e n it follows t h a t g'(x)~/'(x) a.c. This implies t h a t g(x)~/(x), which proves o u r assertion.

Finally, it is easy to verify t h a t / " ( x ) has a j u m p at x =ze/2 § k.~, k = 0,1,2 ... n - 1.

4 2 9

Références

Documents relatifs

3. How many 1:10,000 aerial photographs would be required to give a complete stereo coverage of a rectangular project area 15 km long by 10 km wide? For this project, you are using

which are not independent, what can be done for a solution? Illustrate the situation with three simple equations. 5 8.a) What is the method of separation of variables for

CAUCHY'S THEOREM AND ITS

It was however thought desirable to adhere to the familiar expressions since, as will be shown (a) each &#34;vector&#34; is uniquely determined by certain components, and

LICHTENSTEIN Permanente Bewegungen einer homogenen, i~kompressiblen, ziihen Fliissigkeit [Matematische Zeitschrift, Band 28 (I928), Heft 3, P... LICttTENSTEIN, Uber die

I In the present paper we are concerned with the same special case and employ the like representation though instead of treating our function with

Wenn ich im Einverstandniss mit der Redaction trotzdem meine Tabelle ver- 6ffentliche, so geschieht das, weil R~USCHLE'S Arbeit wenlg bekannt ge- worden und jetzt

off nous supposerons que los coefficients h et k soient des quantit6s ra- tionnelles.. Ddmons~ration du thdor6me fondamcntal de Oalois.. La d&amp;nonstration de la